Question and Answers Forum

All Questions      Topic List

Algebra Questions

Previous in All Question      Next in All Question      

Previous in Algebra      Next in Algebra      

Question Number 172360 by Mikenice last updated on 25/Jun/22

Commented by mr W last updated on 25/Jun/22

(1)  4=(√(16))=(√(13+(√9)))=(√(13+(√(5+4))))=(√(13+(√(5+(√(13+(√(5+(√(13+...))))))))))

$$\left(\mathrm{1}\right) \\ $$$$\mathrm{4}=\sqrt{\mathrm{16}}=\sqrt{\mathrm{13}+\sqrt{\mathrm{9}}}=\sqrt{\mathrm{13}+\sqrt{\mathrm{5}+\mathrm{4}}}=\sqrt{\mathrm{13}+\sqrt{\mathrm{5}+\sqrt{\mathrm{13}+\sqrt{\mathrm{5}+\sqrt{\mathrm{13}+...}}}}} \\ $$

Commented by mr W last updated on 25/Jun/22

(2)  see Q# 170948

$$\left(\mathrm{2}\right) \\ $$$${see}\:{Q}#\:\mathrm{170948} \\ $$

Answered by Rasheed.Sindhi last updated on 26/Jun/22

Let x=(√(13+(√(5+(√(13+(√(5+(√(13+...))))))))))  x^2 −13=(√(5+(√(13+(√(5+(√(13+...))))))))  (x^2 −13)^2 =5+(√(13+(√(5+(√(13+...))))))  (x^2 −13)^2 −5=x  See below the comments of mrW sir↓

$${Let}\:{x}=\sqrt{\mathrm{13}+\sqrt{\mathrm{5}+\sqrt{\mathrm{13}+\sqrt{\mathrm{5}+\sqrt{\mathrm{13}+...}}}}} \\ $$$${x}^{\mathrm{2}} −\mathrm{13}=\sqrt{\mathrm{5}+\sqrt{\mathrm{13}+\sqrt{\mathrm{5}+\sqrt{\mathrm{13}+...}}}} \\ $$$$\left({x}^{\mathrm{2}} −\mathrm{13}\right)^{\mathrm{2}} =\mathrm{5}+\sqrt{\mathrm{13}+\sqrt{\mathrm{5}+\sqrt{\mathrm{13}+...}}} \\ $$$$\left({x}^{\mathrm{2}} −\mathrm{13}\right)^{\mathrm{2}} −\mathrm{5}={x} \\ $$$${See}\:{below}\:{the}\:{comments}\:{of}\:{mrW}\:\boldsymbol{{sir}}\downarrow \\ $$

Commented by mr W last updated on 25/Jun/22

please recheck sir:  (x^2 −13)^2 −5=x  x^4 −26x^2 −x+164=0  (x−4)(x^3 +4x^2 −10x−41)=0  ...

$${please}\:{recheck}\:{sir}: \\ $$$$\left({x}^{\mathrm{2}} −\mathrm{13}\right)^{\mathrm{2}} −\mathrm{5}={x} \\ $$$${x}^{\mathrm{4}} −\mathrm{26}{x}^{\mathrm{2}} −{x}+\mathrm{164}=\mathrm{0} \\ $$$$\left({x}−\mathrm{4}\right)\left({x}^{\mathrm{3}} +\mathrm{4}{x}^{\mathrm{2}} −\mathrm{10}{x}−\mathrm{41}\right)=\mathrm{0} \\ $$$$... \\ $$

Commented by Rasheed.Sindhi last updated on 25/Jun/22

You′re right sir!

$${You}'{re}\:{right}\:\boldsymbol{{sir}}! \\ $$

Commented by mr W last updated on 25/Jun/22

x=(√(13+...))>(√(13))>3.5  all roots of x^3 +4x^2 −10x−41=0   are <3.5, therefore x=4 is the only  suitable solution.

$${x}=\sqrt{\mathrm{13}+...}>\sqrt{\mathrm{13}}>\mathrm{3}.\mathrm{5} \\ $$$${all}\:{roots}\:{of}\:{x}^{\mathrm{3}} +\mathrm{4}{x}^{\mathrm{2}} −\mathrm{10}{x}−\mathrm{41}=\mathrm{0}\: \\ $$$${are}\:<\mathrm{3}.\mathrm{5},\:{therefore}\:{x}=\mathrm{4}\:{is}\:{the}\:{only} \\ $$$${suitable}\:{solution}. \\ $$

Terms of Service

Privacy Policy

Contact: info@tinkutara.com